GET READY FOR THE AMC 12 WITH AoPS
Learn with outstanding instructors and top-scoring students from around the world in our AMC 12 Problem Series online course.
CHECK SCHEDULE

Difference between revisions of "2014 AMC 12B Problems"

(Created page with "==Problem 1== What is <math>10 \cdot \left(\tfrac{1}{2} + \tfrac{1}{5} + \tfrac{1}{10}\right)^{-1}?</math> <math> \textbf{(A)}\ 3\qquad\textbf{(B)}\ 8\qquad\textbf{(C)}\ \frac{2...")
 
(Problem 1)
Line 1: Line 1:
 
==Problem 1==
 
==Problem 1==
What is <math>10 \cdot \left(\tfrac{1}{2} + \tfrac{1}{5} + \tfrac{1}{10}\right)^{-1}?</math>
 
  
<math> \textbf{(A)}\ 3\qquad\textbf{(B)}\ 8\qquad\textbf{(C)}\ \frac{25}{2}\qquad\textbf{(D)}}\ \frac{170}{3}\qquad\textbf{(E)}\ 170</math>
 
  
[[2014 AMC 10A Problems/Problem 1|Solution]]
+
[[2014 AMC 12B Problems/Problem 1|Solution]]

Revision as of 21:53, 8 February 2014

Problem 1

Solution